Help
I could hardly distinguish the difference between (d) and (e). Why is (d) the answer to this ques...
Batman on August 15, 2014
  • June 1997 LSAT
  • SEC1
  • Q13
2
Replies
Help
Could you please explain the reason why (E) shouldn't be the answer? Thanks,
Batman on August 14, 2014
  • June 1997 LSAT
  • SEC1
  • Q8
2
Replies
Setup
Is it possible to view the set up of this game?
MGN2014 on August 14, 2014
  • October 2000 LSAT
  • SEC3
  • Q3
1
Reply
Help
How could both approach be complement each other according to the passage? I don't understand tha...
Batman on August 14, 2014
  • June 1997 LSAT
  • SEC1
  • Q2
2
Replies
Why?
I understand on a sentence level why this sentence could follow. But I am still confused as to wh...
Katie10 on August 13, 2014
  • December 2009 LSAT
  • SEC5
  • Q26
1
Reply
No Answer Choices Showing Up
I don't have any answer choices showing up for this question. Is this happening for anyone else?
CatLSAT on August 8, 2014
  • June 1994 LSAT
  • SEC2
  • Q4
1
Reply
Lines
Where in the passage is the answer?
AglaPkns on July 31, 2014
  • June 2007 LSAT
  • SEC4
  • Q2
1
Reply
Help Plz
Why can't A work? Thank you!
AglaPkns on July 30, 2014
  • June 2007 LSAT
  • SEC2
  • Q2
3
Replies
Question 2
This makes no sense to me ugh...lol.
Ltowns on July 24, 2014
  • December 1992 LSAT
  • SEC2
  • Q2
4
Replies
Why E?
Answer E says "new residents of small cites usually do not pay utilities." The stimulus says "rap...
Derek on July 23, 2014
  • December 2004 LSAT
  • SEC2
  • Q25
1
Reply
Why is this B and not D?
Will you please break this problem down for me.
OffshoreSuge on July 12, 2014
  • October 2011 LSAT
  • SEC3
  • Q18
1
Reply
Help
I picked (d) over (b). I still don't know why (d) can't be an answer and third paragraph is relat...
Batman on July 8, 2014
  • December 1996 LSAT
  • SEC4
  • Q6
2
Replies
Help
Is this "must be true" question? Then, please explain why choice (e) is not the answer. Thanks,
Batman on July 2, 2014
  • October 1996 LSAT
  • SEC4
  • Q10
2
Replies
Help
Could you please illustrate the logical structure of the answer (a) and compare with the stimulus...
Batman on June 8, 2014
  • June 1996 LSAT
  • SEC4
  • Q24
2
Replies
Help
(1) Please explain why choice (c) and (d) can also be "must be true" according to the stimulus. ...
Batman on June 8, 2014
  • October 1996 LSAT
  • SEC1
  • Q7
2
Replies
Help
I don't understand why (c) is the correct answer? Thanks
Batman on June 8, 2014
  • October 1996 LSAT
  • SEC1
  • Q18
2
Replies
Help
I don't understand what the answer (c) means. Is there any proposal on Keziah's argument? What me...
Batman on June 5, 2014
  • June 1996 LSAT
  • SEC4
  • Q2
1
Reply
Help
According to the answer (b), did Eltis really support for part of William's conclusion? Please ex...
Batman on May 31, 2014
  • June 1996 LSAT
  • SEC3
  • Q21
4
Replies
Help
I am totally lost in this question. What does this question mean and why the answer goes to (c)?
Batman on May 31, 2014
  • June 1996 LSAT
  • SEC3
  • Q8
2
Replies
Help
I don't know why answer goes to (d). Pleas explain this. Thanks.
Batman on May 31, 2014
  • June 1996 LSAT
  • SEC3
  • Q7
2
Replies